Question

let x=u+v. y=v find dS, the a vector, and ds^2 for the u,v coordinate system and...

let x=u+v. y=v find dS, the a vector, and ds^2 for the u,v coordinate system and show that it is not an orthogonal system

Homework Answers

Know the answer?
Your Answer:

Post as a guest

Your Name:

What's your source?

Earn Coins

Coins can be redeemed for fabulous gifts.

Not the answer you're looking for?
Ask your own homework help question
Similar Questions
Let u=[6,2 ], v=[3,3 ], and b=[4,1 ]. Find (x⋅u+y⋅v-b)×2 u, where x,y are scalars.
Let u=[6,2 ], v=[3,3 ], and b=[4,1 ]. Find (x⋅u+y⋅v-b)×2 u, where x,y are scalars.
For parts ( a ) − ( c ) , let u = 〈 2 ,...
For parts ( a ) − ( c ) , let u = 〈 2 , 4 , − 1 〉 and v = 〈 4 , − 2 , 1 〉 . ( a ) Find a unit vector which is orthogonal to both u and v . ( b ) Find the vector projection of u onto v . ( c ) Find the scalar projection of u onto v . For parts ( a ) − (...
Let U and V be vector spaces. Show that the Cartesian product U × V =...
Let U and V be vector spaces. Show that the Cartesian product U × V = {(u, v) | u ∈ U, v ∈ V } is also a vector space.
Let vector u= 5i+3j+8k and vector v= i-j+2k Find the component of v parallel to u...
Let vector u= 5i+3j+8k and vector v= i-j+2k Find the component of v parallel to u and the component of v perpendicular to u find a unit vector perpendicular to both u and v
A rectangle with coordinate system with axes x,y,z is rotating relative to an inertial frame with...
A rectangle with coordinate system with axes x,y,z is rotating relative to an inertial frame with constant angular velocity w about the z-axis. A particle of mass m moves under a force whose potential is V(x,y,z). Set up the Lagrange equations of motion in the coordinate system x,y,z. Show that their equations are the same as those in a fixed coordinate system acted on by the force -grad(V) and a force derivable from a velocity dependent potential U. Find U.
It is not true that the equality u x (v x w) = (u x v)...
It is not true that the equality u x (v x w) = (u x v) x w for all vectors. 1. Find explicit vector for u, v and w where this equality does not hold. 2. U, V and W are all nonzero vectors that satisfy the equality. Show that at least one of the conditions below holds: a) v is orthogonal to u and w. b) w is a scalar multiple of u. You can possibly use a...
Let u = ⟨1,3⟩ and v = ⟨4,1⟩. (a) Find an exact expression and a numerical...
Let u = ⟨1,3⟩ and v = ⟨4,1⟩. (a) Find an exact expression and a numerical approximation for the angle between u and v. (b) Find both the projection of u onto v and the vector component of u orthogonal to v. (c) Sketch u, v, and the two vectors you found in part (b).
One electron and two protons in a x,y coordinate system. Find a point with 0 V...
One electron and two protons in a x,y coordinate system. Find a point with 0 V of potential. Fix a coordinate and solve for the other one that gives 0 V.
Compute ∬S (x)(y^2)(z) dS where S is the part of the cone with parameterization r(u,v)= <ucosv,usinv,u>...
Compute ∬S (x)(y^2)(z) dS where S is the part of the cone with parameterization r(u,v)= <ucosv,usinv,u> , 0≤ u ≤ 1, 0≤ v ≤ pi/2 . Also state what the parameter space is.
1. V is a subspace of inner-product space R3, generated by vector u =[2 2 1]T...
1. V is a subspace of inner-product space R3, generated by vector u =[2 2 1]T and v =[ 3 2 2]T. (a) Find its orthogonal complement space V┴ ; (b) Find the dimension of space W = V+ V┴; (c) Find the angle θ between u and v and also the angle β between u and normalized x with respect to its 2-norm. (d) Considering v’ = av, a is a scaler, show the angle θ’ between u and...
ADVERTISEMENT
Need Online Homework Help?

Get Answers For Free
Most questions answered within 1 hours.

Ask a Question
ADVERTISEMENT